Translate

Saturday, June 7, 2014

Số nguyên tố và hợp số

Bài toán 1 (CĐT VMO trường PTNK ĐHQG Tp HCM 2013-2014) 
Cho dãy \left \{ u_n \right \} thỏa mãn \left\{\begin{matrix} u_1=2013\\ u_{n+1}=u_n^3-4u_n^2+5u_n \end{matrix}\right.\forall n\in \mathbb{N}^{*}. Tìm số nguyên tố p thỏa mãn p\equiv 3\;(mod\;4) và p\mid u_{2014}+2009
Lời giải :
Bổ đề quen thuộc : Với các số nguyên dương x,y nguyên tố cùng nhau thì x^2+y^2 không có ước nguyên tố nào dạng 4k+3
Bài toán :
Ta có u_{n+1}=u_n^3-4u_n^2+5u_n-2=(u_n-1)^2(u_n-2)
Do đó
u_{2014}-2=(u_{2013}-1)^2(u_{2013}-2)=(u_{2013}-1)^2(u_{2012}-1)^2(u_{2012}-2)= (u_{2013}-1)^2(u_{2012}-1)^2...(u_1-1)^2 (u_1-2)=2011(u_1-1)^2(u_2-1)^2...(u_{2012}-1)^2(u_{2013}-1)^2
Từ đó suy ra
u_{2014}+2009=2011[(u_1-1)^2(u_2-1)^2...(u_{2012}-1)^2(u_{2013}-1)^2+1]
Theo giả thiết ta có p\equiv 3\;(mod\;4) và p\mid u_{2014}+2009 mà theo bổ đề thì (u_1-1)^2(u_2-1)^2...(u_{2012}-1)^2(u_{2013}-1)^2+1 không có ước nguyên tố nào dạng 4k+3.
Suy ra p\mid 2011\Rightarrow p=2011.
Kết luận : Có duy nhất số nguyên tố p thỏa mãn bài ra là \boxed{p=2011}
*Nhận xét : Các bài toán tương tự :
1) Cho dãy \left \{ u_n \right \} xác định bởi \left\{\begin{matrix} u_1=5\\ u_{n+1}=u_n^3-2u_n+2 \end{matrix}\right.. Tìm số nguyên tố p thỏa mãn p\equiv 3\;(mod\;4)và p\mid u_{2011}+1
2) (THTT số 437) Cho dãy số (a_n) xác định bởi a_1=34 và a_{n+1}=4a_n^3-104a_n^2-107a_n với mọi số nguyên dương n. Tìm tất cả các số nguyên tố p thỏa mãn p\equiv 3\;(mod\;4) và a_{2013}+1 chia hết cho p
Bài toán 2 (Đề nghị Olympic 30-4 toán 10 năm 2011 THPT Hùng Vương, TpHCM) : Tìm các số tự nhiêna,b sao cho A=3^{9a^2+3b-86}+10 là số nguyên tố. 
Lời giải :
Nhận xét rằng phải có 9a^{2}+3b-86\geq 0. Ta luôn có 9a^{2}+3b-86\equiv 1\;\;(mod\;3)
Đặt 9a^{2}+3b-86=3k+1\;\;(k\in \mathbb{N})
Nếu k chẵn thì k=2t\;\;(t\in \mathbb{N}), khi đó 3^{9a^2+3b-86}=3^{3k+1}=3^{6k+1}\equiv 3\;\;(mod\;13)\Rightarrow A\equiv 0\;(mod\;13)
Nhưng do A\in \mathbb{P} nên A=13, suy ra 9a^2+3b-86=1\Leftrightarrow 9a^2+3b=87.
Nhận xét rằng nếu a\geq 4 thì 9a^2+3b\geq 9a^{2}\geq 144>87, suy ra a<4, tức là a\in \left \{ 0,1,2,3\right \}
Thử trực tiếp ta thu được \left ( a,b \right )=(0,29),(1,26),(2,17),(3,2).
Nếu k lẻ thì k=2t+1\;\;(t\in \mathbb{N}), khi đó 3^{9a^2+3b-86}=3^{3k+1}=3^{6t+7}\equiv 3\;\;(mod\;13), lí luận hoàn toàn như trường hợp trên.
Kết luận : \boxed{(a,b)=(0,29),(1,26),(2,17),(3,2)}
Bài toán 3 : Tìm tất cả các số nguyên dương n sao cho luôn tồn tại số nguyên m thỏa mãn 2^n-1\mid m^2+9.
Lời giải :
Bổ đề 1 : Một số nguyên có dạng 4k+3 thì luôn tồn tại một ước số nguyên tố p\equiv 3\;(mod\;4)
Chứng minh bổ đề 1 :
Số nguyên a dạng 4k+3 là một số lẻ nên nó không có ước nguyên tố 2.
Gỉa sử a=p_{1}^{\alpha _1}p_{2}^{\alpha _{2}}...p_{n}^{\alpha _n}\;\;\;(p_i\in \mathbb{P},\alpha _i\in \mathbb{N}^*,i=\overline{1,n})
Nếu p_i\equiv 4\;(mod\;4)\;\forall i=1,2,...,n\Rightarrow a\equiv 1\;(mod\;4) , mâu thuẫn với giả thiết.
Do đó a có ít nhất một ước nguyên tố p\equiv 3\;(mod\;4)
Bổ đề 2 : Nếu các số nguyên x,y và số nguyên tố p\equiv 3\;(mod\;4) thỏa mãn p\mid x^2+y^2 thì p\mid x,p\mid y.
Chứng minh bổ đề 2 :
Nếu p|x hoặc p|y thì hiển nhiên ta có điều phải chứng minh. Xét p\nmid x,p\nmid y
Đặt p=4k+3\qquad(k\in \mathbb{N})
Theo định lí Fermat nhỏ : x^{4k+2}+y^{4k+2}\equiv x^{p-1}+y^{p-1}\equiv 2\;(mod\;p)\qquad(1)
Mặt khác x^{2}+y^2\equiv 0\;(mod\;p)\Rightarrow x^{4k+2}+y^{4k+2}\equiv 0\;(mod\;p)\qquad(2)
Từ (1)(2) suy ra p=2, mâu thuẫn.
Trở lại bài toán :
Nếu n=1 thì hiển nhiên với mọi m nguyên, bài toán đều được thỏa mãn.
Xét n>1. Gọi q là một ước nguyên tố lẻ của n, khi đó 2^{q}-1\equiv 3\;(mod\;4) nên 2^q-1 có ít nhất một ước nguyên tố p\equiv 3\;(mod\;4). Khi đó dễ dàng thấy rằng :
m^2+9\;\vdots \;2^{n}-1\;\vdots \;2^q-1\;\vdots\; p
Khi đó theo bổ đề 2 ta có p|3,p=3. Suy ra 3\mid 2^{q}-1\Rightarrow 2\mid q, điều này mâu thuẫn vì q lẻ.
Như vậy n không có ước nguyên tố lẻ, do đó n=2^k\;(t\in \mathbb{N}^{*}).
Ta sẽ chứng minh rằng với mọi t\in \mathbb{N}^{*} thì n=2^t luôn thỏa mãn đề bài.
Thật vậy,
Ta có 2^n-1=2^{2^{k}}-1=(2^{2^0}+1)(2^{2^1}+1)(2^{2^2}+1)...(2^{2^{t-1}}+1)
Xét hệ đồng dư tuyến tính :
\left\{\begin{matrix} x\equiv 0\;(mod\;2^{2^0}+1)\\ x\equiv 3.2^{2^{0}}\;(mod\;2^{2^1}+1)\\ x\equiv 3.2^{2^{1}}\;(mod\;2^{2^{2}}+1)\\ ...\\ x\equiv 3.2^{2^{k-2}}\;(mod\;2^{2^{k-1}}+1) \end{matrix}\right.
Dễ thấy rằng gcd(2^{2^i}+1,2^{2^j}+1)=1\;\forall i\neq ,j,i,j\in \left \{ 0,1,2,...,k-1 \right \} vì chúng là các số Fermat.
Theo định lí phần dư Trung Hoa thì hệ này chắc chắn có nghiệm x_0.
Ta suy ra
\left\{\begin{matrix} x_{0}^{2}\equiv 0\equiv -9\;(mod\;2^{2^0}+1)\\ x_0^2\equiv 9.2^{2^{1}}\equiv -9\;(mod\;2^{2^1}+1)\\ x_0^2\equiv 9.2^{2^2}\equiv -9\;(mod\;2^{2^2}+1)\\ ...\\ x_0^2\equiv 9.2^{2^{k-1}}\equiv -9\;(mod\;2^{2^{k-1}}+1) \end{matrix}\right.\Rightarrow x_0^2+9\equiv 0\;(mod\;2^n-1)
Khi đó với mọi n=2^k thì luôn tồn tại số nguyên m=x_0 là nghiệm của hệ trên thỏa mãn đề bài.
Kết luận : \boxed{n=2^k\;\;(k\in \mathbb{N})}
Bài toán 3 (Đề thi chọn HSG lớp 12 tỉnh Đồng Nai 2013-2014) : Cho các số nguyên a,b và số nguyên tố p  thỏa mãn \dfrac{a^{2}+b^{2}}{p}\in \mathbb{Z}. Cho biết p là tổng của hai số chính phương. Chứng minh rằng \dfrac{a^{2}+b^{2}}{p}cũng là tổng của hai số chính phương.
 Lời giải :
Đặt p=c^2+d^2 với c,d\in \mathbb{Z}
Ta có \dfrac{a^2+b^2}{p}=\dfrac{(a^2+b^2)(c^2+d^2)}{p^2}=\dfrac{(ad+bc)^2+(ac-bd)^2}{p^2}=\left ( \dfrac{ad+bc}{p} \right )^2+\left ( \dfrac{ac-bd}{p} \right )^2=\left ( \dfrac{ad-bc}{p} \right )^2+\left ( \dfrac{ac+bd}{p} \right )^2
Mặt khác :
(ac+bd)(ac-bd)=a^2c^2-b^2d^2=a^2(c^2+d^2)-d^2(a^2+b^2)=a^2.p+d^2(a+b^2)\vdots p
Nên hoặc p|ac+bd hoặc p|ac-bd
Nếu p|ac+bd thì \dfrac{a^2+b^2}{p}=\left ( \dfrac{ac+bd}{p} \right )^2+\left ( \dfrac{ad-bc}{p} \right )^2
Do \dfrac{a^2+b^2}{p},\dfrac{ac+bd}{p}\in \mathbb{Z}\Rightarrow \dfrac{ad-bc}{p}\in \mathbb{Z}
Suy ra \dfrac{a^2+b^2}{p} là tổng của hai số chính phương.
Trường hợp p|ac-bd tương tự.
Ta có điều phải chứng minh.
Bài toán 4 : Cho các số nguyên tố a,b,c. Đặt x = a+b-c,y=a+c-b,z=b+c-a. Biết rằngx^2=y và hiệu \sqrt{z}-\sqrt{y} là bình phương của một số nguyên tố. Tìm a,b,c
Lời giải :
Đặt
\sqrt{z}-\sqrt{y}=p^{2}\qquad(p\in \mathbb{P})\Leftrightarrow (z+y)-2\sqrt{zy}=p^{4}\Leftrightarrow (a+c-b)+(b+c-a)-2\sqrt{(a+c-b)(b+c-a)}=p^{4}\Leftrightarrow 2c-2\sqrt{(a+c-b)(b+c-a)}=p^{4}\Rightarrow 2|p\Rightarrow p=2
Như vậy ta sẽ giải hệ phương trình sau trên tập nghiệm nguyên tố :
\left\{\begin{matrix} c-\sqrt{(a+c-b)(b+c-a)}=8 & (1)& \\ (a+b-c)^{2}=a+c-b&(2) & \end{matrix}\right.
Từ (1) :
(c-8)^{2}=(a+c-b)(b+c-a)=c^{2}-(a-b)^{2}\Leftrightarrow (c-8)^{2}+(a-b)^{2}=c^{2}
Đây là phương trình Pytagore, hiển nhiên gcd(c-8,a-b,c)=1 do đó nghiệm tổng quát của nó là :
\left\{\begin{matrix} x=2mn & \\ y=m^{2}-n^{2} & \\ z=m^{2}+n^{2} & \end{matrix}\right.
Trong đó m,n\in \mathbb{Z}gcd(m,n)=1m,n khác tính chẵn lẻ.
\blacktriangledown Trường hợp 1 \left\{\begin{matrix} x=c-8=2mn & & \\ y=a-b=m^{2}-n^{2}& & \end{matrix}\right.
Từ hệ dễ dàng suy ra c=2, khi đó từ (2) : (a+b-2)^{2}=a-b+2\qquad(*).
Mặt khác dễ dàng thấy hai số a,b khác tính chẵn lẻ. Do đó a=2 hoặc b=2, thay vào (*) đều không thỏa mãn.
\blacktriangledown Trường hợp 2 : \left\{\begin{matrix} x=a-b=2mn & & \\ y=c-8=m^{2}-n^{2} & & \end{matrix}\right.
Mặt khác, c=m^2+n^2 suy ra m^{2}+n^{2}-8=m^{2}-n^{2}\Rightarrow n=\pm 2.
a) Nếu n=2 : \left\{\begin{matrix} c=m^{2}+4 & & \\ a-b=4m& & \end{matrix}\right.
Thay vào (2) : \left ( a+b-m^{2}-4 \right )^{2}=m^{2}+4+4m=(m+2)^{2}\Leftrightarrow a+b-m^{2}-4=m+2\vee a+b-m^{2}-4=-m-2
  • Trường hợp a+b-m^{2}-4=m+2 , ta có a-b=4m. Từ đó suy ra \left\{\begin{matrix} a=\dfrac{(m+2)(m+3)}{2} & & \\ b=\dfrac{m^{2}-3m+6}{2}& & \end{matrix}\right.
Vì gcd(m,n)=gcd(m,2)=1 nên m lẻ. Đặt m=2k+1\qquad(k\in \mathbb{Z}). Ta có a=\dfrac{(m+2)(m+3)}{2}=\dfrac{(2k+3)(2k+4)}{2}=(2k+3)(k+2).
Vì a nguyên tố nên 2k+3=\pm 1 hoặc k+2=\pm 1. Ta thấy k=-3 thỏa mãn vì khi đó a=3. Từ đó m=-5,b=23,c=29
  • Trường hợp a+b-m^{2}-4=-m-2 , ta có a-b=4m. Từ đó \left\{\begin{matrix} a=\dfrac{(m+1)(m+2)}{2} & & \\ b=\dfrac{m^{2}-5m+2}{2} & & \end{matrix}\right.
Tương tự trên ta thấy m lẻ, đặt m=2k+1\qquad(k\in \mathbb{Z}). Khi đó a=\dfrac{(m+1)(m+2)}{2}=\dfrac{(2k+2)(2k+3)}{2}=(k+1)(2k+3).
Vì a nguyên tố nên 2k+3=\pm 1 hoặc k+1=\pm 1. Mọi giá trị của k tìm được đều không thỏa mãn
b) Nếu n=-2. Giải tương tự trường hợp a.
Kết luận \boxed{(a;b;c)=(3;23;29)}
Bài toán 5 (Đề nghị Olympic 30-4 toán 10 2010 THPT Chuyên Vị Thanh tỉnh Hậu Giang)
 Tìm tất cả các số nguyên dương n sao cho phần nguyên của biểu thức A=\dfrac{n^{3}+8n^{2}+1}{3n} là một số nguyên tố
Lời giải :
Ta có A=\dfrac{n^{3}+8n^{2}+1}{3n}=\dfrac{n^{2}}{3}+2n+\dfrac{2n}{3}+\dfrac{1}{3n}
  • Trường hợp 1 : Xét n = 3k (k\in \mathbb{Z}^{+}) :
A=\dfrac{(3k)^{2}}{3}+2.(3k)+\dfrac{2.(3k)}{3}+\dfrac{1}{3.(3k)}=3k^{2}+8k+\dfrac{1}{9k}
Do đó \left [ A \right ]=3k^{2}+8k=k(3k+8)
Để \left [ A \right ]\in \mathbb{P}  thì k = 1
  • Trường hợp 2 : Xét n = 3k+1 (k\in \mathbb{N})
A=\dfrac{(3k+1)^{2}}{3}+2(3k+1)+\dfrac{2(3k+1)}{3}+\dfrac{1}{3(3k+1)}=3k^{2}+10k+3+\dfrac{1}{9k+3}
Do đó \left [ A \right ]=3k^{2}+10+3=(k+3)(3k+1)
Để \left [ A \right ]\in \mathbb{P} thì k=0
  • Trường hợp 3 : Xét n = 3k+2 (k\in \mathbb{N})
A=\dfrac{(3k+2)^{2}}{3}+2(3k+2)+\dfrac{2(3k+2)}{3}+\dfrac{1}{3(3k+2)}=3k^{2}+12k+6+\dfrac{2}{3}+\dfrac{1}{3(3k+2)}
Hiển nhiên \dfrac{1}{3(3k+2)}<\dfrac{1}{3}\Rightarrow \dfrac{2}{3}+\dfrac{1}{3(3k+2)}<1\Rightarrow \left [ A \right ]=3k^{2}+12k+6
Do 3|\left [ A \right ] và \left [ A \right ]>3 nên \left [ A \right ] là hợp số.
Kết luận : \boxed{n\in \left \{ 1;3 \right \}}
Nhận xét : Ta xét số dư của số n khi chia cho 3 là vì mẫu số là một biểu thức chia hết cho 3.
Bài toán 6  (IMO 2001) :  Cho bốn số nguyên a,b,c,d thỏa mãn điều kiện :
\left\{\begin{matrix} a>b>c>d>0 & & \\ ac+bd=(b+d+a-c)(b+d-a+c)& & \end{matrix}\right.
Chứng minh rằng ab + cd là hợp số.
Lời giải :
Gỉa thiết đã cho tương đương :
ac+bd=(b+d+a-c)(b+d-a+c)=(b+d)^{2}-(a-c)^{2}=(b^{2}+d^{2}+2bd)-(a^{2}+c^{2}-2ac)\Leftrightarrow a^{2}-ac+c^{2}=b^{2}+bd+d^{2}
Do đó :
(ab+cd)(ad+bc)=bd(a^{2}+c^{2})+ac(b^{2}+d^{2})=bd(a^{2}-ac+c^{2})+ac(b^{2}+bd+d^{2})=(ac+bd)(a^{2}-ac+c^{2})\qquad(*)
Chú ý rằng với điều kiện a>b>c>d>0 ta có :
\left\{\begin{matrix} (ab+cd)-(ac+bd)=(a-d)(b-c) >0& & \\ (ac+bd)-(ad+bc)=(a-b)(c-d)>0& & \end{matrix}\right.
\Rightarrow ab+cd>ac+bd>ad+bc
Gỉa sử ab + cd là một số nguyên tố, vì ab + cd > ac+bd nên gcd(ab+cd,ac+bd)=1
Mặt khác, từ (*) ta có :
(ac+bd)|(ab+cd)(ad+bc) vì gcd(ab+cd,ac+bd)=1 nên (ac+bd)|(ad+bc).
Điều này là vô lí vì ac + bd > ad + bc > 0
Vậy : Gỉa thiết phản chứng sai, ab + cd phải là hợp số
Bài toán 7 : Tìm bảy số nguyên tố sao cho tổng các lũy thừa bậc 6 của chúng bằng tích của chúng.
Lời giải :
Gọi bảy số nguyên tố cần tìm là p_{1},p_{2},...,p_{7}.
Theo đề bài ta có p_{1}^{6}+p_{2}^{6}+...+p_{7}^{6}=p_{1}p_{2}...p_{7}
Gỉa sử tất cả các số nguyên tố p_{1},p_{2},...,p_{7} đều không chia hết cho 7.
Theo định lí Fermat nhỏ ta có p_{i}^{6}\equiv 1(mod7)\Rightarrow \sum_{i=1}^{7}p_{i}^{6}\equiv 0(mod7)\Rightarrow p_{1}p_{2}...p_{7}\equiv 0(mod7)
Điều này là vô lí vì 7\nmid p_{i}
Do đó ít nhất một trong các số nguyên tố p_{1},p_{2},...,p_{7} chia hết cho 7, giả sử 7|p_{7}\Rightarrow p_{7}=7
Khi đó ta lại có p_{1}^{6}+...+p_{6}^{6}+7^{6}=7p_{1}p_{2}...p_{6}
Lại giả sử p_{1},p_{2},...,p_{6} đều không chia hết cho 7.
\Rightarrow p_{i}^{6}\equiv 1(mod7)\Rightarrow \sum_{i=1}^{6}p_{i}^{6}\equiv 6(mod7)
Điều này là vô lí vì \sum_{i=1}^{6}p_{i}^{6}=7p_{1}p_{2}...p_{6}-7^{6}\equiv 0(mod7)
Suy ra ít nhất một trong các số p_{1},p_{2},...,p_{6} phải chia hết cho 7. Gỉa sử 7|p_{6}\Rightarrow p_{6}=7
Tiếp tục như vậy, cuối cùng ta có p_{i}=7
Kết luận : \boxed{p_{1}=p_{2}=...=p_{7}=7} 
Bài toán 8 : Cho a,b,c là ba số nguyên khác 0 và a\neq c sao cho \dfrac{a}{c}=\dfrac{a^{2}+b^{2}}{c^{2}+b^{2}}. Chứng minh rằng a^{2}+b^{2}+c^{2} không phải là một số nguyên tố. 
Lời giải :
Từ giả thiết \dfrac{a}{c}=\dfrac{a^{2}+b^{2}}{c^{2}+a^{2}}\Rightarrow ac^{2}+ab^{2}=a^{2}c+b^{2}c\Rightarrow (ac-b^{2})(c-a)=0
Vì a\neq c nên phải có ac=b^{2}
\blacktriangledown Trường hợp 1 : Nếu gcd(a,c)=d>1\qquad(d\in \mathbb{N})
Đặt \left\{\begin{matrix} a=dx & & \\ c=dy& & \end{matrix}\right.\qquad(x,y\in \mathbb{Z},gcd(x,y)=1)
Do đó a^{2}+b^{2}+c^{2}=a^{2}+ac+c^{2}=d^{2}(x^{2}+xy+y^{2})
Dễ thấy d^{2}>1 và d^{2} không thể là một số nguyên tố.
Do đó trường hợp này ta có a^{2}+b^{2}+c^{2} là hợp số
\blacktriangledown Trường hợp 2 : Nếu gcd(a,c)=1 thì vì ac=b^{2}\Rightarrow \left\{\begin{matrix} a=m^{2} & & \\ c=n^{2}& & \end{matrix}\right.\vee \left\{\begin{matrix} a=-m^{2} & & \\ c=-n^{2}& & \end{matrix}\right.\left ( m,n\in \mathbb{Z} \right )
Trong cả hai trường hợp ta đều có :
a^{2}+b^{2}+c^{2}=a^{2}+ac+c^{2}=m^{4}+m^{2}n^{2}+n^{4}=(m^{2}+n^{2}-mn)(m^{2}+n^{2}+mn)
Mặt khác dễ thấy m^2+mn+n^2>m^2-mn+n^2>1 nên trường hợp này ta cũng có a^2+b^2+c^2 là hợp số
Vậy : a^{2}+b^{2}+c^{2} không là số nguyên tố.

No comments: